Đến nội dung

DarkBlood nội dung

Có 618 mục bởi DarkBlood (Tìm giới hạn từ 09-05-2020)



Sắp theo                Sắp xếp  

#475801 Xác định vị trí đường thẳng $d$ để chu vi tứ giác $BDEC$...

Đã gửi bởi DarkBlood on 06-01-2014 - 20:55 trong Hình học

Cho tam giác $ABC.$ Đường thẳng $d$ quay quanh $A,$ $d$ không cắt đoạn thẳng $BC.$ $D$ và $E$ lần lượt là hình chiếu của $B$ và $C$ trên $d.$ Xác định vị trí đường thẳng $d$ để chu vi tứ giác $BDEC$ lớn nhất.




#482515 Xác định vị trí của điểm $A$ để: $\frac{MA.MB.MC(MA+...

Đã gửi bởi DarkBlood on 11-02-2014 - 11:22 trong Hình học

Cho $\Delta ABC$ cân đỉnh $A$ nội tiếp đường tròn $(O;R)$, $M$ là điểm trên cung $BC$ không chứa điểm $A$.

Xác định vị trí của điểm $A$ để: $\frac{MA.MB.MC(MA+MB+MC)}{MA.MB+MB.MC+MC.MA}$ đạt $GTLN$

Phải là xác định điểm $M$ chứ nhỉ?

 

Áp dụng định lý Ptoleme, ta có $AB.MC+AC.MB=MA.BC$ hay $AB(MB+MC)=MA.BC$

 

Ta có

 

$\bullet\ MA.MB+MB.MC+MC.MA=MA(MB+MC)+MB.MC= \\ =\dfrac{(BM+CM)^2.AB}{BC}+MB.MC\geq MB.MC.\left ( \dfrac{4AB}{BC}+1 \right )$

 

$\bullet\ MA(MA+MB+MC)=MA\left ( MA+\dfrac{MA.BC}{AB} \right )= \\ =MA^2\left ( \dfrac{BC}{AB}+1 \right )\leq 4R^2.\left ( \dfrac{BC}{AB}+1 \right )$

Do đó

$\dfrac{MA.MB.MC(MA+MB+MC)}{MA.MB+MB.MC+MC.MA}\leq \dfrac{MB.MC.4R^2.\left ( \dfrac{BC}{AB}+1 \right )}{MB.MC.\left ( \dfrac{4AB}{BC}+1 \right )} \\ \Leftrightarrow \dfrac{MA.MB.MC(MA+MB+MC)}{MA.MB+MB.MC+MC.MA}\leq 4R^2.\dfrac{BC\left (AB+BC  \right )}{AB\left (4AB+BC  \right )}=\textrm{const}$

 

Vậy $\frac{MA.MB.MC(MA+MB+MC)}{MA.MB+MB.MC+MC.MA}$ đạt giá trị lớn nhất khi $MA$ đi qua $O,$ $MB=MC$ $\Leftrightarrow$ $M$ là điểm chính giữa cung $BC.$




#383059 Xác định $M,$ $N$ để độ dài đoạn thẳng $MN$ nhỏ...

Đã gửi bởi DarkBlood on 02-01-2013 - 21:29 trong Hình học

Câu 1, giả thiết cho không chặt, $M,N$ trùng với $A$ là xong
Câu 2: $AF.FC = FI.FE = AF^2$

Câu 1: $M$ thuộc $BC$ mà anh, sao mà trùng $A$ được :mellow:

__
@BlackSel: trùng với C, anh nhầm.



#383038 Xác định $M,$ $N$ để độ dài đoạn thẳng $MN$ nhỏ...

Đã gửi bởi DarkBlood on 02-01-2013 - 21:13 trong Hình học

Câu 1: Cho tam giác đều $ABC$. $M,$ $N,$ là các điểm lần lượt chuyển động trên hai cạnh $BC$ và $AC$ sao cho $BM=CN.$ Xác định vị trí của $M,$ $N$ để độ dài đoạn $MN$ nhỏ nhất.
Câu 2: Cho hình vuông $ABCD.$ Trên cạnh $BC$ lấy điểm $E$ tùy ý $(E\neq B;$ $E\neq C).$ Kẻ tia $Ax$ vuông góc với $AE,$ tia $Ax$ cắt đường thẳng $CD$ tại $F.$ Trung tuyến $AI$ của tam giác $AEF$ cắt $CD$ tại $K,$ đường thẳng kẻ qua $E$ song song với $AB$ cắt $AI$ ở $G.$ Chứng minh $AF^2=FK.FC$
____________________
P/s: Mọi người gợi ý cho em thôi nha! :)



#506467 Với $a,b,c \geq0$, chứng minh rằng: $a+b+c\geq 3...

Đã gửi bởi DarkBlood on 14-06-2014 - 00:06 trong Bất đẳng thức và cực trị

Với $a,b,c \geq0$, chứng minh rằng:

$a+b+c\geq 3\sqrt[3]{abc}$

p/s: Chứng minh theo BĐT Cô-si cho 2 số.

BĐT tưởng đương $a+b+c+\sqrt[3]{abc}\geq 4\sqrt[3]{abc}$

Áp dụng BĐT Cô si 2 số 

$a+b\geq 2\sqrt{ab}$

$c+\sqrt[3]{abc}\geq 2\sqrt{c\sqrt[3]{abc}}$

$\sqrt{ab}+\sqrt{c\sqrt[3]{abc}}\geq 2\sqrt[4]{abc\sqrt[3]{abc}}=2\sqrt[3]{abc}$

Từ đó có điều phải chứng minh.




#370179 Vậy trong bao nhiêu phút thì An sẽ vượt được anh của An?

Đã gửi bởi DarkBlood on 17-11-2012 - 21:30 trong Các dạng toán khác

Gọi quãng đường từ nhà đến trường là $s$.
Vận tốc của An là $v_1$, vận tốc của anh An là $v_2$.
Ta có: $v_1=\frac{s}{30}$
$v_2=\frac{s}{40}$
Sau khi anh An đi được $5$ phút thì An bắt đầu khởi hành, như vậy lúc đó anh của An cách An:
$s'=v_2.5=\frac{s}{40}.5=\frac{s}{8}$
Thời gian để An vượt anh An là:
$t=\frac{s'}{v_1-v_2}=\frac{\frac{s}{8}}{\frac{s}{30}-\frac{s}{40}}=15$ (phút).



#498717 Trận 9 - Bất đẳng thức

Đã gửi bởi DarkBlood on 13-05-2014 - 08:14 trong Thi giải toán Marathon cấp THCS 2014

Em nghĩ những bất đẳng thức như cô si 3 số, bunhiacopxki, BCS không có trong chương trình học cấp 2 nên cần phải chứng minh lại ạ.




#498752 Trận 9 - Bất đẳng thức

Đã gửi bởi DarkBlood on 13-05-2014 - 11:27 trong Thi giải toán Marathon cấp THCS 2014

Sách NCPT đã giới thiệu rồi mà sao phải chứng minh lại?

Chương trình cấp 2 là trong sách giáo khoa mà. 




#498132 Trận 9 - Bất đẳng thức

Đã gửi bởi DarkBlood on 09-05-2014 - 21:56 trong Thi giải toán Marathon cấp THCS 2014

Cho $x, y, z$ là các số dương thỏa mãn điều kiện $xyz = 1$. Tìm giá trị nhỏ nhất của biểu thức:  

$$E=\frac{1}{x^3(y+z)}+\frac{1}{y^3(z+x)}+\frac{1}{z^3(x+y)}.$$

Toán thủ ra đề: angleofdarkness

Bài làm của MSS 10

Bổ đề 1: Với mọi số thực $a, b, c, x, y, z$ $(x, y, z>0)$ ta luôn có:

$$\dfrac{a^2}{x}+\dfrac{b^2}{y}+\dfrac{c^2}{z}\geq \dfrac{(a+b+c)^2}{x+y+z}$$

Chứng minh

Trước hết ta chứng minh với mọi số thực $m, n, p, q$ $(p, q>0)$ ta có

$$\dfrac{m^2}{p}+\dfrac{n^2}{q}\geq \dfrac{(m+n)^2}{p+q} \ \ \ \ (\star)$$

Thật vậy bất đẳng thức $(\star)$ tương đương với $\dfrac{(mq-np)^2}{pq(p+q)}\geq 0$ $($Luôn đúng vì $p, q>0)$

Đẳng thức xảy ra $\Leftrightarrow \dfrac{m}{p}=\dfrac{n}{q}$

Áp dụng bất đẳng thức $(\star),$ ta có

$$\dfrac{a^2}{x}+\dfrac{b^2}{y}+\dfrac{c^2}{z}\geq \dfrac{(a+b)^2}{x+y}+\dfrac{c^2}{z}\geq \dfrac{(a+b+c)^2}{x+y+z}$$

Vậy ta có điều phải chứng minh. Đẳng thức xảy ra $\Leftrightarrow \dfrac{a}{x}=\dfrac{b}{y}=\dfrac{c}{z}$

 

Bổ đề 2: Với mọi số dương $a, b, c$ ta có

$$a^3+b^3+c^3\geq 3abc$$

Chứng minh

Bất đẳng thức đã cho tương đương với $\dfrac{1}{2}(a+b+c)[(a-b)^2+(b-c)^2+(c-a)^2]\geq 0$ (Luôn đúng với $a,b,c >0)$

Đẳng thức xảy ra $\Leftrightarrow a=b=c.$

 

Quay lại bài toán

Đặt $x=\dfrac{1}{a}, y=\dfrac{1}{b}, z=\dfrac{1}{c}$ $(a,b,c>0)$

 

Vì $xyz=1$ nên $abc=1$

 

Khi đó $$\textrm{E}=\dfrac{a^3bc}{b+c}+\dfrac{ab^3c}{c+a}+\dfrac{abc^3}{a+b}=\dfrac{a^2}{b+c}+\dfrac{b^2}{c+a}+\dfrac{c^2}{a+b}$$

Áp dụng bổ đề 1, ta có: $$\textrm{E}\geq \dfrac{(a+b+c)^2}{2(a+b+c)}=\dfrac{a+b+c}{2}$$

Áp dụng bổ đề 2, ta có: $$\textrm{E}=\dfrac{a+b+c}{2}\geq\dfrac{3\sqrt[3]{abc}}{2}=\dfrac{3}{2}$$

Đẳng thức xảy ra $\Leftrightarrow a=b=c=1 \Leftrightarrow x=y=z=1$

 

Vậy $\textrm{min}\ \textrm{E}=\dfrac{3}{2}$ khi $x=y=z=1.$

 

d = 10

S = 56




#498155 Trận 9 - Bất đẳng thức

Đã gửi bởi DarkBlood on 09-05-2014 - 23:10 trong Thi giải toán Marathon cấp THCS 2014

Cho $x, y, z$ là các số dương thỏa mãn điều kiện $xyz = 1$. Tìm giá trị nhỏ nhất của biểu thức:  

$$E=\frac{1}{x^3(y+z)}+\frac{1}{y^3(z+x)}+\frac{1}{z^3(x+y)}.$$

Toán thủ ra đề: angleofdarkness

Mở rộng 1 của MSS 10

Cho số nguyên dương $n$ và các số dương $x, y, z$ thỏa mãn điều kiện $xyz=1.$ Tìm giá trị nhỏ nhất của biểu thức:

$$\textrm{A}=\dfrac{1}{x^{n+1}(y+z)}+\dfrac{1}{y^{n+1}(z+x)}+\dfrac{1}{z^{n+1}(x+y)}$$

 

Bài làm

 

Bổ đề 1: $($đã chứng minh$)$ Với mọi số thực $a, b, c, x, y, z$ $(x, y, z>0)$ ta luôn có:

$$\dfrac{a^2}{x}+\dfrac{b^2}{y}+\dfrac{c^2}{z}\geq \dfrac{(a+b+c)^2}{x+y+z}$$

 

Bồ đề 2: $($đã chứng minh$)$ Với mọi số dương $a, b, c,$ ta có

$$a^3+b^3+c^3\geq 3abc$$

 

Bổ đề 3: Với mọi số nguyên dương $n$ và các số dương $a, b, c,$ ta có

$$\dfrac{a^n}{b+c}+\dfrac{b^n}{c+a}+\dfrac{c^n}{a+b}\geq \dfrac{a^{n-1}+b^{n-1}+c^{n-1}}{2}$$

Chứng minh

Theo bổ đề 1 ta có $$\sum \dfrac{a^n}{b+c}=\sum \dfrac{a^{2(n-1)}}{a^{n-2}(b+c)}\geq \dfrac{(a^{n-1}+b^{n-1}+c^{n-1})^2}{\sum a^{n-2}(b+c)}$$

Đặt $n-2=k.$

Như vậy cần chứng minh $$\dfrac{a^{n-1}+b^{n-1}+c^{n-1}}{\sum a^{n-2}(b+c)}\geq \dfrac{1}{2}$$

$$\Leftrightarrow \dfrac{a^{k+1}+b^{k+1}+c^{k+1}}{\sum a^{k}(b+c)}\geq \dfrac{1}{2}$$

$$\Leftrightarrow 2(a^{k+1}+b^{k+1}+c^{k+1})\geq \sum a^{k}(b+c)$$

$$\Leftrightarrow (a^k-b^k)(a-b)+(a^k-c^k)(a-c)+(b^k-c^k)(b-c)\geq 0$$

Nếu $b\geq c>0$ thì $b^k-c^k\geq 0$ và $b-c\geq 0,$ suy ra $(b^k-c^k)(b-c)\geq 0$

 

Nếu $c\geq b>0$ thì $b^k-c^k\leq 0$ và $b-c\leq 0,$ suy ra $(b^k-c^k)(b-c)\geq 0$

 

Vậy $(b^k-c^k)(b-c)\geq 0$ với mọi $b, c>0$

 

Không mất tổng quát giả sử $a=\textrm{max}\left \{ a;\ b;\ c \right \}$

 

Suy ra $(a^k-b^k)(a-b)\geq 0$ và $(a^k-c^k)(a-c)\geq 0.$

 

Do đó $(a^k-b^k)(a-b)+(a^k-c^k)(a-c)+(b^k-c^k)(b-c)\geq 0$

 

Vậy ta có điều phải chứng minh. Đẳng thức xảy ra $\Leftrightarrow a=b=c.$

 

Quay lại bài toán

Đặt $x=\dfrac{1}{a}, y=\dfrac{1}{b}, z=\dfrac{1}{c}$ $(a,b,c>0)$

 

Vì $xyz=1$ nên $abc=1$

 

Khi đó $$\textrm{A}=\dfrac{a^{n+1}bc}{b+c}+\dfrac{ab^{n+1}c}{c+a}+\dfrac{abc^{n+1}}{a+b}=\dfrac{a^n}{b+c}+\dfrac{b^n}{c+a}+\dfrac{c^n}{a+b}$$

Áp dụng bổ đề 3, ta có $$\textrm{A}\geq \dfrac{a^{n-1}+b^{n-1}+c^{n-1}}{2}$$

Áp dụng bổ đề 2, ta có $$\textrm{A}\geq \dfrac{3\sqrt[3]{a^{n-1}.b^{n-1}.c^{n-1}}}{2}=\dfrac{3}{2}$$

Vậy $\textrm{min}\ \textrm{A}=\dfrac{3}{2}$ khi $a=b=c=1\Leftrightarrow x=y=z=1.$




#492295 Trận 7 - PT, HPT đại số

Đã gửi bởi DarkBlood on 11-04-2014 - 22:06 trong Thi giải toán Marathon cấp THCS 2014

Giải phương trình: $2x^{2}+5x-1=7\sqrt{x^{3}-1}}\ (\star)$

Đề thi của l4lzTeoz

Bài làm của MSS10: Em xin gửi lại bài làm

Cách 1:

ĐKXĐ: $x\geq 1$

 

Đặt $\sqrt{x-1}=a,\ \sqrt{x^2+x+1}=b.$ $(a\geq 0, b>0)$

 

Khi đó $\left\{\begin{matrix} 3a^2+2b^2=3x-3+2x^2+2x+2=2x^2+5x-1\\ ab=\sqrt{(x-1)(x^2+x+1)}=\sqrt{x^3-1} \end{matrix}\right.$

 

Phương trình đã cho trở thành

 

$3a^2+2b^2=7ab$

 

$\Leftrightarrow (3a-b)(a-2b)=0$

 

$\Leftrightarrow \begin{bmatrix} 3a=b\\ a=2b \end{bmatrix}$

 

 

Nếu $3a=b,$ ta có phương trình

 

$3\sqrt{x-1}=\sqrt{x^2+x+1}$

 

$\Leftrightarrow 9x-9=x^2+x+1$

 

$\Leftrightarrow x=4\pm\sqrt{6}\ (\textrm{TM})$

 

 

Nếu $a=2b,$ ta có phương trình

 

$\sqrt{x-1}=2\sqrt{x^2+x+1}$

 

$\Leftrightarrow x-1=4x^2+4x+4$ $($phương trình vô nghiệm vì $\Delta =-71<0)$

 

 

Vậy tập nghiệm của phương trình $(\star)$ là

 

$\boxed{S=\left \{ 4+\sqrt{6}\ ;\ 4+\sqrt{6} \right \}}$

 

 

Cách 2:

ĐKXĐ: $x\geq 1$

Ta có

$2x^{2}+5x-1=7\sqrt{x^{3}-1}$

$\Rightarrow (2x^2+5x-1)^2-49(x^3-1)=0$

$\Leftrightarrow (x^2-8x+10)(4x^2+3x+5)=0$

$\Leftrightarrow \begin{bmatrix} x^2-8x+10=0\\ 4x^2+3x+5=0 \end{bmatrix}$

 

Trường hợp 1: 

$x^2-8x+10=0$

$\Leftrightarrow x=4\pm\sqrt{6}\ (\textrm{TM})$

 

 

 

Trường hợp 2:

$4x^2+3x+5=0$ $($phương trình vô nghiệm vì $\Delta =-71<0)$

 

Vậy tập nghiệm của phương trình $(\star)$ là

$\boxed{S=\left \{ 4+\sqrt{6}\ ;\ 4+\sqrt{6} \right \}}$

 

  Hoan nghênh giải nhiều cách nhưng điểm vẫn vậy




#492276 Trận 7 - PT, HPT đại số

Đã gửi bởi DarkBlood on 11-04-2014 - 21:35 trong Thi giải toán Marathon cấp THCS 2014

Giải phương trình: $2x^{2}+5x-1=7\sqrt{x^{3}-1}\ (\star)$

Đề thi của l4lzTeoz

Bài làm của MSS10:

ĐKXĐ: $x\geq 1$

Đặt $\sqrt{x-1}=a,\ \sqrt{x^2+x+1}=b.$ $(a\geq 0, b>0)$

Khi đó $\left\{\begin{matrix} 3a^2+2b^2=3x-3+2x^2+2x+2=2x^2+5x-1\\ ab=\sqrt{(x-1)(x^2+x+1)}=\sqrt{x^3-1} \end{matrix}\right.$

Phương trình đã cho trở thành

$3a^2+2b^2=7ab$

$\Leftrightarrow (3a-b)(a-2b)=0$

$\Leftrightarrow \begin{bmatrix} 3a=b\\ a=2b \end{bmatrix}$

 

Nếu $3a=b,$ ta có phương trình

$3\sqrt{x-1}=\sqrt{x^2+x+1}$

$\Leftrightarrow 9x-9=x^2+x+1$

$\Leftrightarrow 4\pm\sqrt{6}\ (\textrm{TM})$

 

Nếu $a=2b,$ ta có phương trình

$\sqrt{x-1}=2\sqrt{x^2+x+1}$

$\Leftrightarrow x-1=4x^2+4x+4$ $($phương trình vô nghiệm vì $\Delta =-71<0)$

 

Vậy tập nghiệm của phương trình $(\star)$ là

$\boxed{S=\left \{ 4+\sqrt{6}\ ;\ 4+\sqrt{6} \right \}}$

 

 

      d =10

      S =17+10.3=47




#489501 Trận 6 - Phương trình nghiệm nguyên, đồng dư, chia hết

Đã gửi bởi DarkBlood on 29-03-2014 - 21:19 trong Thi giải toán Marathon cấp THCS 2014

Tồn tại hay không các cặp số nguyên (x,y) thỏa mãn phương trình sau đây ?

$$\sqrt{2025x^2+2012x+3188}=2013x-2011y+2094\ \ \ \ \ \ \ (1)$$

Đề của 

lenin1999

Bài làm của MSS 10:

Vì $x, y$ nguyên nên $2013x-2011y+2094 \in \mathbb{Z}$ 

Do đó $\sqrt{2025x^2+2012x+3188} \in \mathbb{N}$ $($vì $\sqrt{2025x^2+2012x+3188}\geq 0)$ 

 

Đặt $\sqrt{2025x^2+2012x+3188}=k$ $(k\in \mathbb{N})$

Khi đó $2025x^2+2012x+3188=k^2$

$\Leftrightarrow (2025x+1006)^2-(45k)^2=-5443664$

$\Leftrightarrow (2025x-45k+1006)(2025+45k+1006)=-5443664=-2^4.397.857\ \ \ \ \ \ (2)$

 

Vì $x \in \mathbb{Z}$ và $k \in \mathbb{N}$ nên $2025x-45k+1006\ ; 2025x+45k+1006 \in \mathbb{Z}$

Và $2025x-45k+1006\leq 2025x+45k+1006$

Mặt khác $(2025x-45k+1006)+(2025x+45k+1006)$ chia hết cho $2$ nên $2025x-45k+1006$ và $2025x+45k+1006$ cùng tính chẵn lẻ.

Lại có $2025x-45k+1006$ và $2025x-45k+1006$ đều chia $15$ dư $1$

 

Do đó phương trình $(1)$ tương đương với

$\left\{\begin{matrix} 2025x-45k+1006=-794\\ 2025x+45k+1006=6856 \end{matrix}\right. \Leftrightarrow \left\{\begin{matrix} x=1\\ k=85 \end{matrix}\right.$
 
Thay $x=1$ vào phương trình $(1)$ ta có phương trình
$\sqrt{2025+2012+3188}=2013-2011y+2094\Leftrightarrow y=2$
 
Vậy phương trình có nghiệm nguyên
$$(x;y)=(1;2)$$
 
 
   d =10
  S =17+10x3=47



#485732 Trận 4 - Bất đẳng thức

Đã gửi bởi DarkBlood on 03-03-2014 - 23:13 trong Thi giải toán Marathon cấp THCS 2014

Hình như những mở rộng của bạn buiminhhieu không liên quan tới bài toán thì phải  :mellow:




#485215 Trận 4 - Bất đẳng thức

Đã gửi bởi DarkBlood on 28-02-2014 - 22:13 trong Thi giải toán Marathon cấp THCS 2014

 

Cho $x,y\in \mathbb{R}$ thỏa mãn $(x+y)^{3}+4xy\geq 2$ . Tìm giá trị nhỏ nhất của:

$$ P=3(x^{4}+y^{4}+x^{2}y^{2})-2(x^{2}+y^{2})+1$$

Đề của 

nk0kckungtjnh

Bài làm của $MSS 10$

Áp dung bất đẳng thức $(a+b)^2\geq 4ab\ (a, b\in \mathbb{R})$ $($bất đẳng thức tương đương $(a-b)^2\geq 0,$ luôn đúng$),$ ta có: 

$2\leq (x+y)^3+4xy\leq (x+y)^3+(x+y)^2$

 

Do đó $(x+y)^3+(x+y)^2-2\geq 0\Leftrightarrow (x+y-1)[(x + y)^2+2(x+y)+2]\geq 0$

 

Mà $(x+y)^2+2(x+y)+2=(x+y+1)^2+1>0$ với mọi $x, y$

 

Nên $x+y-1\geq 0$ hay $x+y\geq 1$

 

Lại có $(x+y)^2\leq 2(x^2+y^2)$ $($bất đẳng thức tương đương $(x-y)^2\geq 0,$ luôn đúng$)$

 

Do đó $x^2+y^2\geq \dfrac{1}{2}$

 

 

Ta có: $P=3(x^4+y^4+x^2y^2)-2(x^2+y^2)+1=3(x^2+y^2)^2-3x^2y^2-2(x^2+y^2)+1$

 

Mà $x^2y^2\leq \dfrac{1}{4}(x^2+y^2)^2$

 

Nên $P\geq 3(x^2+y^2)^2-\dfrac{3}{4}(x^2+y^2)^2-2(x^2+y^2)+1$

 

$\Leftrightarrow P\geq \dfrac{9}{4}\left [ \left ( x^2+y^2 \right )-\dfrac{4}{9} \right ]^2+\dfrac{5}{9}$

 

Vì $x^2+y^2\geq \dfrac{1}{2}$

 

Nên $P\geq \dfrac{9}{4}\left ( \dfrac{1}{2}-\dfrac{4}{9} \right )^2+\dfrac{5}{9}=\dfrac{9}{16} $($Vì $

 

\dfrac{1}{2}-\dfrac{4}{9}>0)$

 

Đẳng thức xảy ra $\Leftrightarrow \left\{\begin{matrix} (x+y)^3+4xy=2\\ x=y\\ x+y=1 \end{matrix}\right.\Leftrightarrow x=y=\dfrac{1}{2}$

 

 

 

Vậy $\textrm{min}\ P=\dfrac{9}{16}$ khi $ x=y=\dfrac{1}{2}$

 

Điểm 10 .




#485493 Trận 4 - Bất đẳng thức

Đã gửi bởi DarkBlood on 02-03-2014 - 17:45 trong Thi giải toán Marathon cấp THCS 2014

Cho $x,y\in \mathbb{R}$ thỏa mãn $(x+y)^{3}+4xy\geq 2$ . Tìm giá trị nhỏ nhất của:

$$ P=3(x^{4}+y^{4}+x^{2}y^{2})-2(x^{2}+y^{2})+1$$

Đề của 

nk0kckungtjnh

MSS 10

 

Mở rộng 1: Cho $x,y,z \in \mathbb{R}$ thỏa mãn $(x+y+z)^{3}+3(xy+yz+zx)\geq 2$ . Tìm giá trị nhỏ nhất của:

$$P=9(x^{4}+y^{4}+z^{4}+x^{2}y^{2}+y^{2}z^{2}+z^{2}x^{2})-3(x^{2}+y^{2}+z^{2})+2$$

 

Lời giải

Ta có $2\leq (x+y+z)^2+3(xy+yz+zx)\leq (x+y+z)^3+(x+y+z)^2$ 

Nên $(x+y+z)^3+(x+y+z)^2-2\geq 0 \Leftrightarrow (x+y+z-1)\left [(x+y+z)^2+2(x+y+z)+2 \right ]\geq 0$

Mà $(x+y+z)^2+2(x+y+z)+2>0$ nên $x+y+z-1\geq 0 \Leftrightarrow x+y+z\geq 1$

Mặt khác $x^2+y^2+z^2\geq \dfrac{1}{3}(x+y+z)^2$ nên $x^2+y^2+z^2\geq \dfrac{1}{3}.$

 

Ta có:

$P=9(x^{4}+y^{4}+z^{4}+x^{2}y^{2}+y^{2}z^{2}+z^{2}x^{2})-3(x^{2}+y^{2}+z^{2})+2$

$\Leftrightarrow P=9(x^2+y^2+z^2)^2-9(x^2y^2+y^2z^2+z^2x^2)-3(x^2+y^2+z^2)+2$

Mà $x^2y^2+y^2z^2+z^2x^2\leq \dfrac{1}{3}(x^2+y^2+z^2)^2$

Nên $P\geq 6(x^2+y^2+z^2)^2-3(x^2+y^2+z^2)+2=6\left [ (x^2+y^2+z^2)-\dfrac{1}{4} \right ]^2+\dfrac{13}{8}$

Vì $x^2+y^2+z^2\geq \dfrac{1}{3}$ nên $P\geq 6\left ( \dfrac{1}{3}-\dfrac{1}{4} \right )^2+\dfrac{13}{8}=\dfrac{5}{3}$

 

Vậy $\textrm{min}\ P=\dfrac{5}{3}$ khi $x=y=z=\dfrac{1}{3}.$

$$\star \star \star \star \star \star \star \star \star \star$$

Mở rộng 2: Cho $x,y\in \mathbb{R}$ thỏa mãn $(x+y)^{3}+4xy\geq 2$. Tìm giá trị nhỏ nhất của biểu thức sau với $n$ $(n\in \mathbb{N}^*)$ 

$$P=4\left ( x^{2^{n+1}}+y^{2^{n+1}}+x^{2^n}y^{2^n} \right )-\dfrac{6}{2^{2^n}}\left ( x^{2^n}+y^{2^n} \right )$$

 

Lời giải

Bổ đề Cho $x,y\in \mathbb{R}$ thỏa mãn $x+y\geq 1$ thì $x^{2^n}+y^{2^n}\geq \dfrac{1}{2^{2^{n}-1}}$ với mọi $n\in \mathbb{N}^*$ $\left ( \star \right )$

Chứng minh 

Với $n=1,$ ta có: $x^2+y^2\geq \dfrac{(x+y)^2}{2}\geq \dfrac{1}{2}=\dfrac{1}{2^{2^{1}-1}}$

Giả sử $\left ( \star \right )$ đúng với $n=k$ 

Khi đó $x^{2^k}+y^{2^k}\geq \dfrac{1}{2^{2^{k}-1}}$

Ta chứng minh với $n=k+1$ thì

$x^{2^{k+1}}+y^{2^{k+1}}\geq \dfrac{1}{2^{2^{k+1}-1}}$

Thật vậy, ta có

$x^{2^{k+1}}+y^{2^{k+1}}=\left ( x^{2^k} \right )^2+\left ( y^{2^k} \right )^2\geq \dfrac{1}{2}\left ( x^{2^k}+y^{2^k} \right )^2\geq \dfrac{1}{2}\left ( \dfrac{1}{2^{2^{k}-1}} \right )^2$

Mà $2\left (2^{2^{k}-1} \right )^2=2^{2^{k+1}-1}$ nên $x^{2^{k+1}}+y^{2^{k+1}}\geq \dfrac{1}{2^{2^{k+1}-1}}$

Vậy theo nguyên lý quy nạp, bổ đề được chứng minh.

 

Quay lại bài toán, từ lời giải bài toán gốc, ta được $x+y\geq 1$

Do đó $x^{2^n}+y^{2^n}\geq \dfrac{1}{2^{2^{n}-1}}$

Tương tự bài toán gốc, ta chứng minh được

$P\geq 3\left ( x^{2^n}+y^{2^n} \right )^2-\dfrac{6}{2^{2^n}}\left ( x^{2^n}+y^{2^n} \right )$

$\Leftrightarrow P\geq 3\left ( x^{2^n}+y^{2^n}-\dfrac{1}{2^{2^n}} \right )^2-\dfrac{3}{2^{2^{n+1}}}$

Mà $x^{2^n}+y^{2^n}\geq \dfrac{1}{2^{2^{n}-1}}$

Nên $P\geq 3\left ( \dfrac{1}{2^{2^{n}-1}}-\dfrac{1}{2^{2^n}} \right )^2-\dfrac{3}{2^{2^{n+1}}}=0$

 

Vậy $\textrm{min}\ P=0$ khi $x=y=\dfrac{1}{2}$

$$\star \star \star \star \star \star \star \star \star \star$$

Mở rộng 3: Giải hệ phương trình

$$\left\{\begin{matrix} x^3+y^3+3xy(x+y+2)=x^2+y^2+2 \\ 3(x^4+y^4+x^2y^2)−2(x^2+y^2)+\dfrac{7}{16}=0 \end{matrix}\right.$$

 

Lời giải

Từ phương trình ban đầu, ta có:

$(x+y)^3+4xy=2+(x-y)^2\geq 2$

Do đó $3(x^4+y^4+x^2y^2)−2(x^2+y^2)+1\geq \dfrac{9}{16}$ hay $3(x^4+y^4+x^2y^2)−2(x^2+y^2)+\dfrac{7}{16}\geq 0$

Đẳng thức xảy ra khi $x=y=\dfrac{1}{2}$

Do đó hệ phương trình có nghiệm $x=y=\dfrac{1}{2}.$

 

Có 4 nhận xét cho 3 bài mở rộng.

1) Anh ủng hộ em tìm tòi ra nhiều hướng như thế , nhưng .... ( đọc 2,3,4 )

2) MR 1 thì em không nên mở rộng bằng cách tạo ra thêm 1 ẩn ( nếu $n$ ẩn thì ok ) . Hãy tôn trọng ý tưởng người ra đề .

3 ) MR2 thì sai hoàn toàn . Định nghĩa hàm mũ là cơ số phải dương khác 1 .

4 ) Như 2 , em nên tông trọng dạng bài của tác giả

 

Điểm MR 0đ.




#489020 Trận 3 - Hình học

Đã gửi bởi DarkBlood on 27-03-2014 - 11:26 trong Thi giải toán Marathon cấp THCS 2014

Bài này cũng bị lỗi hình vẽ mà anh 

BlackSelena

Diễn đàn hình như bị lỗi, em nộp lại hình cho bài của mình và anh tính điểm nha.
(Trừ nửa số điểm thì to quá @@)
Hình 1:

Hình 2:

 

Bài mình hiện giờ không có hình là do trang upanh mới ngừng hoạt động, còn lúc anh BlackSelena chấm bài thì vẫn có hình nha bạn :)




#483179 Trận 3 - Hình học

Đã gửi bởi DarkBlood on 15-02-2014 - 00:07 trong Thi giải toán Marathon cấp THCS 2014

Do đề của các Toán thủ nộp chưa phù hợp nên trận này BTC sẽ ra đề

Đề của BTC:
Cho tam giác $ABC$ nhọn. $D,E,F$ trên $BC, CA, AB$ sao cho $\triangle DEF$ nhọn và $AD, BE, CF$ đồng quy. $M, N, P$ trên $EF, FD, DE$ sao cho $\triangle MNP$ nhọn và $DM, EN, FP$ đồng quy.

Chứng minh rằng: $AM, BN, CP$ cũng đồng quy.

Bài làm của MSS 10:

Bổ đề 1: Định lý Menelaus

Bổ đề 2: Định lý Ceva

Bổ đề 3: Cho tam giác $ABC,$ một đường thẳng $d$ cắt cạnh $AB, AC$ lần lượt tại $D$ và $E.$ Lấy $N, Q$ thuộc $BC.$ Gọi $M, P$ lần lượt là giao điểm của $AN, AQ$ với $DE.$ Khi đó ta có $\dfrac{NB}{NC}=\dfrac{BQ}{CQ}\cdot \dfrac{DM}{EM}\cdot \dfrac{PE}{PD}$

Chứng minh: 

58724256.bode.png

Qua $E$ kẻ đường thẳng song song với $BC$ cắt $AB, AN, AQ$ lần lượt tại $F, I, K.$ 

Áp dụng định lý Talet vào 2 tam giác $ABN\ (FI\parallel BN)$ và $ACN\ (EI\parallel CN),$ suy ra:

$$\dfrac{BN}{FI}=\dfrac{CN}{EI}\left ( =\dfrac{NA}{IA} \right )\Leftrightarrow \dfrac{BN}{CN}=\dfrac{FI}{EI}$$

Chứng minh tương tự, ta có $\dfrac{BQ}{CQ}=\dfrac{FK}{EK}$

Như vậy cần chứng minh $$\dfrac{FI}{EI}=\dfrac{KF}{KE}\cdot \dfrac{DM}{EM}\cdot \dfrac{PE}{PD}$$

hay $$\dfrac{DM}{EM}\cdot \dfrac{EI}{FI}=\dfrac{KE}{KF}\cdot \dfrac{PD}{PE}$$

Áp dụng định lý Menelaus cho tam giác $DEF$ cát tuyến $AMI,$ ta có

$$\dfrac{DM}{EM}\cdot \dfrac{EI}{FI}\cdot \dfrac{FA}{DA}=1\Leftrightarrow \dfrac{DM}{EM}\cdot \dfrac{EI}{FI}=\dfrac{DA}{FA}$$

Áp dụng định lý Menelaus cho tam giác $DEF$ cát tuyến $APK,$ ta có

$$\dfrac{KE}{KF}\cdot \dfrac{FA}{DA}\cdot \dfrac{PD}{EP}=1\Leftrightarrow \dfrac{KE}{KF}\cdot \dfrac{PD}{EP}=\dfrac{DA}{FA}$$

Do đó $$\dfrac{DM}{EM}\cdot \dfrac{EI}{FI}=\dfrac{KE}{KF}\cdot \dfrac{PD}{PE}$$

Vậy bổ đề được chứng minh.

 

Quay lại bài toán.

58724254.bailam.png

Gọi $M_1, N_1, P_1$ lần lượt là giao điểm $AM, BN, CP$ với $BC, CA, AB.$

Gọi $M_2, N_2, P_2$ lần lượt là giao điểm $AD, BE, CF$ với $EF, FD, DE.$

Áp dụng định lý Ceva cho tam giác $ABC$ có $AD, BE, CF$ đồng quy, ta có

$$\dfrac{BD}{CD}\cdot \dfrac{CE}{AE}\cdot \dfrac{AF}{BF}=1$$

Áp dụng định lý Ceva cho tam giác $DEF$ có $DM, EN, FP$ đồng quy, ta có

$$\dfrac{FM}{EM}\cdot \dfrac{DN}{FN}\cdot \dfrac{EP}{DP}=1$$

Áp dụng định lý Ceva cho tam giác $DEF$ có $DM_2, EN_2, FP_2$ đồng quy $($do $AD, BE, CF$ đồng quy$)$ , ta có

$$\dfrac{M_2E}{M_2F} \cdot \dfrac{N_2F}{N_2D} \cdot \dfrac{P_2D}{P_2E}=1$$

Áp dụng bổ đề 3, ta có:

$$\dfrac{M_1B}{M_1C}=\dfrac{BD}{CD}\cdot \dfrac{FM}{EM}\cdot \dfrac{M_2E}{M_2F}$$

$$\dfrac{N_1C}{N_1A}=\dfrac{CE}{AE}\cdot \dfrac{DN}{FN}\cdot \dfrac{N_2F}{N_2D}$$

$$\dfrac{P_1A}{P_1B}=\dfrac{AF}{BF}\cdot \dfrac{EP}{DP}\cdot \dfrac{P_2D}{P_2E}$$

Do đó $$\dfrac{M_1B}{M_1C}\cdot \dfrac{N_1C}{N_1A}\cdot \dfrac{P_1A}{P_1B}=\left ( \dfrac{BD}{CD}\cdot \dfrac{CE}{AE}\cdot \dfrac{AF}{BF} \right ) \left ( \dfrac{FM}{EM}\cdot \dfrac{DN}{FN}\cdot \dfrac{EP}{DP} \right ) \left ( \dfrac{M_2E}{M_2F} \cdot \dfrac{N_2F}{N_2D} \cdot \dfrac{P_2D}{P_2E} \right )$$

$$\Leftrightarrow \dfrac{M_1B}{M_1C}\cdot \dfrac{N_1C}{N_1A}\cdot \dfrac{P_1A}{P_1B}=1$$

Theo định lý Ceva, ta có $AM_1, BN_1, CP_1$ đồng quy, tức là $AM, BN, CP$ đồng quy. 

 

$d = 10$

$S = 46$




#478872 Trận 2 - PT, HPT

Đã gửi bởi DarkBlood on 24-01-2014 - 23:31 trong Thi giải toán Marathon cấp THCS 2014

Rất xin lỗi các toán thủ đã vì post đề chậm trễ, sau đây là đề thi trận 2 MSS:

Đề của toán thủ : Best Friend

 

$$\left\{\begin{matrix} 8x^{2}+12y^{2}-20xy=0 & & \\ 4x^{2}-6x+1=y^{2}-3y & & \end{matrix}\right.$$

Thời gian làm bài tính từ: 23h ngày 24/1/2014

Bài làm của DarkBlood:

Ta có hệ

$\left\{\begin{matrix} 8x^{2}+12y^{2}-20xy=0 & & \\ 4x^{2}-6x+1=y^{2}-3y & & \end{matrix}\right.$

tương đương với

$\left\{\begin{matrix} (2x-3y)(x-y)=0  & & \\ 4x^{2}-6x+1=y^{2}-3y\ (\star) & & \end{matrix}\right.$

 

Trường hợp 1: $2x-3y=0\Leftrightarrow y=\dfrac{2}{3}x$

Thay vào $(\star),$ ta có:

$4x^2-6x+1=\dfrac{4}{9}x^2-2x$

$\Leftrightarrow 32x^2-36x+9=0$

$\Leftrightarrow \begin{bmatrix} x=\dfrac{3}{4}\\ x=\dfrac{3}{8} \end{bmatrix}$

Với $x=\dfrac{3}{4}$ thì $y=\dfrac{1}{2}$

Với $x=\dfrac{3}{8}$ thì $y=\dfrac{1}{4}$

 

Trường hợp 2: $x-y=0\Leftrightarrow x=y$

Thay vào $(\star),$ ta có: 

$4x^{2}-6x+1=x^{2}-3x$

$\Leftrightarrow 3x^2-3x+1=0$ $($Phương trình vô nghiệm vì $3x^2-3x+1=3\left ( x-\dfrac{1}{2} \right )^2+\dfrac{1}{4}>0\ \forall x)$

 

Vậy phương trình có nghiệm $\left ( x;y \right )\in \left \{ \left ( \dfrac{3}{4};\dfrac{1}{2} \right );\left ( \dfrac{3}{8};\dfrac{1}{4} \right ) \right \}.$

_________________________

$d = 10$
$S = 46$




#501092 Trận 10 - Bất đẳng thức

Đã gửi bởi DarkBlood on 23-05-2014 - 22:53 trong Thi giải toán Marathon Chuyên toán 2014

Bài vừa rồi em gửi nhầm ạ, BQT đừng chấm.




#501085 Trận 10 - Bất đẳng thức

Đã gửi bởi DarkBlood on 23-05-2014 - 22:33 trong Thi giải toán Marathon Chuyên toán 2014

Cho 3 số dương $x,y,z$ thỏa mãn $xy+yz+xz=3xyz$. Chứng minh rằng : 

$$\sum \frac{x^2y^2}{(x^2+y^2)(1+xy)} \leq \frac{3}{4}$$

Đề của 

Twisted Fate

Bài làm của MSS 10

Bổ đề 1: Với mọi số thực dương $a, b$ ta có $\dfrac{4}{a+b}\leq \dfrac{1}{a} + \dfrac{1}{b}$

Chứng minh

Bất đẳng thức tương đương $(a-b)^2\geq 0$ $($Đúng$)$

Đẳng thức xảy ra $\Leftrightarrow a=b$

 

Bổ đề 2: Với mọi số thực $a,b,c$ ta có $ab+bc+ca\leq a^2+b^2+c^2$

Chứng minh

Bất đẳng thức tương đương $(a-b)^2+(b-c)^2+(c-a)^2\geq 0$ $($Đúng$)$

Đẳng thức xảy ra $\Leftrightarrow a=b=c$

 

Trở lại bài toán

Ta có $xy+yz+zx=3xyz \Leftrightarrow \dfrac{1}{x}+\dfrac{1}{y}+\dfrac{1}{z}=3$

 

Đặt $\dfrac{1}{x}=a, \dfrac{1}{y}=b, \dfrac{1}{z}=c$ thì $a+b+c=3,\ a,b,c>0$

 

Khi đó, bất đẳng thức đã cho trở thành

$$\sum \dfrac{ab}{(a^2+b^2)(ab+1)} \leq \dfrac{3}{4}$$

Áp bổ đề 1, ta có:

$$\sum \dfrac{ab}{(a^2+b^2)(ab+1)} \leq \dfrac{1}{4} \left ( \sum \dfrac{ab}{a^2+b^2}+\sum \dfrac{ab}{ab+1} \right )$$

Áp dụng bất đẳng thức cauchy cho 2 số dương, ta có

$$\sum \dfrac{ab}{a^2+b^2}+\sum \dfrac{ab}{ab+1}\leq \sum \dfrac{ab}{2ab}+\sum \dfrac{ab}{2\sqrt{ab}}=\dfrac{3}{2}+\sum \dfrac{\sqrt{ab}}{2}$$

Như vậy cần chứng minh

$$\sum \sqrt{ab} \leq 3$$ 

Theo bổ đề 2, ta có: $\sum \sqrt{ab} \leq \sum a=3$

 

Vậy ta có điều phải chứng minh. Đẳng thức xảy ra $\Leftrightarrow a=b=c=1 \Leftrightarrow x=y=z=1$




#476850 trên mặt bàn có 2005 như nhau

Đã gửi bởi DarkBlood on 12-01-2014 - 10:26 trong Các dạng toán khác

trên mặt bàn có 2005 như nhau, mỗi đồng có 1 mặt xanh và 1 mặt đỏ và khi bắt đầu toàn bộ mặt xanh đều ngửa lên. mỗi một lần ta đổi mặt 4 đồng xu trên mặt bàn. hỏi sau 2006 lượt có thể để tất cả đồng xu mặt đỏ hướng lên đc ko.

b) tổng quát bài toán 

Tổng quát: Trên măt bàn có $2k+1$ $(k\in \mathbb{N}^*),$ mỗi đồng có $1$ mặt xanh và $1$ mặt đỏ và khi bắt đầu toàn bộ mặt xanh đều ngửa lên. Mỗi một lần ta đổi mặt $2n$ $(n\in \mathbb{N}^*\ ;\ n\leq k)$ đồng xu trên mặt bàn. Hỏi sau $m$ $(m\in \mathbb{N}^*)$ lượt có thể để tất cả đồng xu mặt đỏ hướng lên được không?

 

Thay mỗi đồng xu mặt đỏ là số $0,$ mỗi đồng xu mặt xanh là số $1.$ 

Thao tác thực hiện mỗi lần là thay $a$ số $0$ thành số $1,$ thay $b$ số $1$ thành số $0$ $(a+b=2n\ ;\ a,b \in \mathbb{N})$

Do đó tổng các số sẽ tăng $a$ đơn vị và giảm $b=2n-a$ đơn vị sau mỗi thao tác, nghĩa là tăng (giảm) $a-(2n-a)=2(a-n)$ đơn vị.

Vì vậy tính chẵn lẻ của tổng các số không thay đổi sau mỗi thao tác.

Mà tổng các số ban đầu là số lẻ $($Vì ban đầu có $2k+1$ số $1$ và $0$ số $0)$

Nên sau $m$ lần thực hiện thao tác tổng các số vẫn là số lẻ, hay sau $m$ lần thực hiện thì luôn tồn tại ít nhất $1$ số $1.$ 

Vậy sau $m$ lần thực hiện trên bàn luôn tồn tại ít nhất $1$ đồng xu mặt xanh, không thể để tất cả đồng xu mặt đỏ đều hướng lên được.




#390090 Topic: Các bài toán về tính chia hết

Đã gửi bởi DarkBlood on 25-01-2013 - 22:45 trong Số học

3. Tìm tất cả các số tự nhiên $n$ để $2^{n}-1$ chia hết cho $7$. CMR với mọi stn n thì $2^{n}+1$ không chia hết cho 7

$a)$
Xét $n=3k$ $(k\in N),$ ta có:
$2^n-1=2^{3k}-1=8^k-1=7A$ $\vdots$ $7$
Xét $n=3k+1$ $(k\in N),$ ta có:
$2^n-1=2^{3k+1}-1=8^k.2-1=(7B+1).2-1=7B+1$ $($loại$)$

Xét $n=3k+2$ $(k\in N),$ ta có:
$2^n-1=2^{3k+2}-1=8^k.4-1=(7B+1).4-1=7B+3$ $($loại$)$
Vậy với mọi số tự nhiên $n$ chia hết cho $3$ thì $2^{n}-1$ chia hết cho $7$.

$b)$ Ta có:
$2^{n}+1=2^{n}-1+2$
Theo câu $a$ thì với mọi số tự nhiên $n$ thì $2^{n}-1$ chia cho $7$ có số dư là $0;$ $1$ hoặc $3$
Do đó $2^{n}-1+2$ chia cho $7$ có số dư là $2;$ $3$ hoặc $5$
Hay $2^{n}+1$ chia cho $7$ có số dư là $2;$ $3$ hoặc $5$
Vậy với mọi số tự nhiên $n$ thì $2^{n}+1$ không chia hết cho $7.$




1. Chứng minh các biểu thức sau nguyên với mọi n nguyên:
b,$\frac{n^{9}}{630}-\frac{n^{7}}{21}+\frac{13n^{5}}{30}-\frac{82n^{2}}{63}+\frac{32n}{35}$

Sai đề nhé, lấy máy tính thử $n=0$ sẽ thấy :)



#411087 Topic: Các bài toán về tính chia hết

Đã gửi bởi DarkBlood on 07-04-2013 - 17:44 trong Số học

 Tìm số tự nhiên $n$ để $3^{2n}+3^{n}+1$ chia hết cho 13.

Đặt $A=3^{2n}+3^n+1.$ 

 

Xét $n=3k$ $(k\in \mathbb{N}),$ ta có:

$A=3^{6k}+3^{3k}+1=(3^3)^{2k}+(3^3)^k+1$

Mà $3^3\equiv 1\ (\bmod\ 13)$ nên $A=(3^3)^{2k}+(3^3)^k+1\equiv 1+1+1=3\ (\bmod\ 13)$

 

Xét $n=3k+1$ $(k\in \mathbb{N}),$ ta có:

$A=3^{6k+2}+3^{3k+1}+1=(3^3)^{2k}.9+(3^3)^k.3+1$

Mà $3^3\equiv 1\ (\bmod\ 13)$ nên $A=(3^3)^{2k}.9+(3^3)^k.3+1\equiv 9+3+1\equiv 0\ (\bmod\ 13)$

 

Xét $n=3k+2$ $(k\in \mathbb{N}),$ ta có:

$A=3^{6k+4}+3^{3k+2}+1=(3^3)^{2k+1}.3+(3^3)^k.9+1$

Mà $3^3\equiv 1\ (\bmod\ 13)$ nên $A=(3^3)^{2k+1}.3+(3^3)^k.9+1\equiv 3+9+1\equiv 0\ (\bmod\ 13)$

 

Vậy với mọi số tự nhiên $n$ không chia hết cho $3$ thì $3^{2n}+3^n+1$ luôn chia hết cho $13.$




#384277 Topic: Các bài toán về tính chia hết

Đã gửi bởi DarkBlood on 06-01-2013 - 21:30 trong Số học

Mọi người có cần bài không ?
- Nếu $a+b+c$ chia hết cho 6 thì $a^{3}+b^{3}+c^{3}$ chia hết cho 6.
- $2009^{2010}$ không chia hết cho 2010 .
- $n^{2}+7n+22$ không chia hết cho 9 .

$a)$Ta có:
$a^3+b^3+c^3-a-b-c=(a^3-a)+(b^3-b)+(c^3-c)=(a-1)a(a+1)+(b-1)b(b+1)+(c-1)c(c+1)$
Vì $a+b+c$ $\vdots $ $6$ và $(a-1)a(a+1)+(b-1)b(b+1)+(c-1)c(c+1)$ $\vdots $ $6$ nên $a^3+b^3+c^3$ $\vdots $ $6$.

$b)$ $2009^{2010}$ không chia hết cho 2 nên $2009^{2010}$ không chia hết cho $2010$

$c)$ Giả sử $n^2+7n+22$ chia hết cho $9,$ suy ra $n^2+7n+22$ chia hết cho $3$.
Ta có:
$(n+5)^2=n^2+10n+25=n^2+7n+22+3(n+1)$
Mà $n^2+7n+22$ chia hết cho $3$ nên $(n+5)^2$ chia hết cho $3$.
Do đó $n+5$ chia hết cho $3$.
Suy ra $n=3k-5$ $(t\in Z)$
Ta có:
$n^2+7n+22=(3k-5)^2+7(3k-5)+22$
$=9k^2-39k+25+21k-35+22=9k^2-9k+9+3,$
không chia hết cho 3, trái với điều giả sử.
Vậy $n^{2}+7n+22$ không chia hết cho $9$ .